GMAT Verbal : Standard Inference Logic

Study concepts, example questions & explanations for GMAT Verbal

varsity tutors app store varsity tutors android store

Example Questions

Example Question #8 : Inference Critical Reasoning

A television news network has recently been criticized for failing to give as much time to individuals who do not believe climate change is occurring as they do to scientists who believe climate change is occurring, even though the network does give equal time to all sides of the debates over immigration, tax reform, and gun policy. The network claims that they only give equal time to both sides of a debate when one side cannot be definitively proven by existing scientific research.

Which of the following can be correctly inferred from the information given above?

Possible Answers:

The television news network believes that it is important to avoid debate on scientific discoveries.

If the news network gave the same time it gave to scientists to individuals who don’t believe in climate change, it would increase its ratings.

There are no effective counter arguments against climate change that might sway viewers to believe that climate change is not occurring.

The television news network believes that the existence of climate change has been definitely proven by existing scientific research.

No individuals who spoke on the network and who did not believe climate change was occurring were scientists.

Correct answer:

The television news network believes that the existence of climate change has been definitely proven by existing scientific research.

Explanation:

Whenever you are asked to make an inference from an argument presented on the GMAT, remember that inferences don't need to be interesting or surprising - they only need to be guaranteed.

The argument here states that a television network has been criticized for not giving as much time to climate change deniers as it does to those who believe in climate change even though they do give equal time to all sides of the debates surrounding other issues like tax reform and immigration. The network claims that this is because they only give equal time if one side of the debate cannot be definitively proven by science.

Since the network does not give equal time to both sides of the climate change debate, that means that people at the network believe that it fits the exception given and that one side (the side that believes in climate change) has been definitively proven by science, which matches answer choice "The television news network believes that the existence of climate change has been definitely proven by existing scientific research.".

Among the other answers, "The television news network believes that it is important to avoid debate on scientific discoveries." can be discarded because while the network does not give equal time for all sides of the debate in some cases, it does not provide a blanket dismissal of debating all scientific discovery. Choice "There are no effective counter arguments against climate change that might sway viewers to believe that climate change is not occurring." can be eliminated since while the network believes the science has been settled, this is not the same thing as claiming that no one will be swayed by the arguments against climate change. Choice "No individuals who spoke on the network and who did not believe climate change was occurring were scientists." is a bit harder to eliminate since the stimulus tells you that the people who believe that climate change is occurring are scientists, but it doesn't specifically say that the individuals who don't believe in climate change aren't scientists, so "No individuals who spoke on the network and who did not believe climate change was occurring were scientists." can be eliminated. Choice "If the news network gave the same time it gave to scientists to individuals who don’t believe in climate change, it would increase its ratings." can also be eliminated since even though some people criticize the network for its current policy, it is unknown how a change in policy would affect overall criticism of the network (and in turn how that would effect ratings).

Example Question #9 : Inference Critical Reasoning

A computer equipped with fingerprint recognition software, which denies access to a computer to anyone whose fingerprint is not on file, identifies a person's fingerprint by analyzing not only the detailed structure of the fingerprint, but also such characteristics as the level of pressure upon which the finger is placed on the scanner and the finger's skin tone. Even the most adept computer hackers cannot duplicate all the characteristics the software analyzes.

Which of the following can be logically concluded from the passage above?

Possible Answers:

Computers equipped with the recognition software will soon be installed in most financial firms that deal with sensitive electronic information.

It is not possible for any top computer hacker to gain access to a computer equipped with the recognition software solely by virtue of skill in replicating the structure of fingerprints.

The fingerprint recognition software is so sensitive that many authorized users are often denied legitimate access.

Fingerprint recognition software has taken many years and tremendous investment to develop and perfect.

Use of the recognition software is largely impractical due to the time it takes to record and analyze a fingerprint.

Correct answer:

It is not possible for any top computer hacker to gain access to a computer equipped with the recognition software solely by virtue of skill in replicating the structure of fingerprints.

Explanation:

The correct answer to this question is "It is not possible for any top computer hacker to gain access to a computer equipped with the recognition software solely by virtue of skill in replicating the structure of fingerprints.". This is an INFERENCE question, requiring the test taker to choose the correct answer that must be true based on the information provided in the stimulus. "Use of the recognition software is largely impractical due to the time it takes to record and analyze a fingerprint." is incorrect as the passage provides no information with regard to the speed of recording and analyzing the fingerprint; as such, no related conclusion can be drawn. "Computers equipped with the recognition software will soon be installed in most financial firms that deal with sensitive electronic information." is incorrect as the passage provides no information with regard to the installation of computers that possess the software in specific locations; as such, no related conclusion can be drawn. "It is not possible for any top computer hacker to gain access to a computer equipped with the recognition software solely by virtue of skill in replicating the structure of fingerprints." This is the correct answer. The passage states that the software detects more characteristics than those that the most successful hackers are able to duplicate; as such, we can conclude it would be impossible for any top hacker to gain access to a protected computer solely by replicating one of multiple characteristics analyzed by the software. "Fingerprint recognition software has taken many years and tremendous investment to develop and perfect." is incorrect as the passage provides no information with regard to the time and investment costs associated with the development of the software; as such, no related conclusion can be drawn. "The fingerprint recognition software is so sensitive that many authorized users are often denied legitimate access." is incorrect as the passage provides no information with regard to errors produced by the software; as such, no related conclusion can be drawn.

Example Question #10 : Inference Critical Reasoning

If the minimum wage increases again, MacDowell’s will have to increase the prices it charges for its products. And if that happens, MacDowell’s has a choice: it can spend more on advertising to attract more customers, or its sales and profitability will decrease. But since the extra advertising costs will simply raise total expenses, increasing those costs will still result in an overall decrease in profitability.

Which one of the following conclusions can be logically drawn from the statements above?

Possible Answers:

MacDowell’s will see its profitability increase if the minimum wage does not increase.

MacDowell’s will be unable to maintain its current profitability if the minimum wage increases.

Unless the minimum wage increases, MacDowell’s will continue to remain profitable.

If MacDowell’s sees a reduction in its profitability, that means that the minimum wage has increased.

If the minimum wage increases, MacDowell’s will no longer be able to remain profitable.

Correct answer:

MacDowell’s will be unable to maintain its current profitability if the minimum wage increases.

Explanation:

Because this is an Inference question, the degree of proof for the correct answer is that the correct answer MUST BE TRUE. Because of that:

Choice "Unless the minimum wage increases, MacDowell’s will continue to remain profitable." is incorrect because you're not told what happens if the minimum wage does not increase. This prediction is hard to make, then: suppose the minimum wage stayed flat but a disease was traced to MacDowell's ingredients or a fire burned down its top-grossing store. There are plenty of ways for profitability to be cut even if the minimum wage stays flat.

Choice "If the minimum wage increases, MacDowell’s will no longer be able to remain profitable." is incorrect because it goes too far. You know that profitability will decrease, but not that it will go away entirely.

Choice "MacDowell’s will see its profitability increase if the minimum wage does not increase." is incorrect for similar reasons to choice "Unless the minimum wage increases, MacDowell’s will continue to remain profitable.". There are plenty of factors aside from the minimum wage that could decrease profitability, so choice "MacDowell’s will see its profitability increase if the minimum wage does not increase." is not necessarily true.

Choice "MacDowell’s will be unable to maintain its current profitability if the minimum wage increases." is correct. Because you're told in the argument that, of the two options that would face MacDowell's in the event of a minimum wage increase, both will decrease profitability, you know it to be true that a wage increase will cut profitability.

Choice "If MacDowell’s sees a reduction in its profitability, that means that the minimum wage has increased." is incorrect for similar reasons to "Unless the minimum wage increases, MacDowell’s will continue to remain profitable." and "MacDowell’s will see its profitability increase if the minimum wage does not increase.". Plenty of other factors can lead to a decrease in profitability, so that decrease on its own does not allow you to infer that it was specific to a minimum wage hike.

Example Question #1 : Standard Inference Logic

Most pain relievers come with warnings against continuous use longer than 7 consecutive days. While some people might be able to safely use a particular pain reliever for a longer period of time, many people will begin to experience side effects if the warnings are ignored.

The information above most strongly supports which of the following?

Possible Answers:

Anyone who wants to maximize their natural health and well-being should avoid pain relievers entirely.

Any side effects experienced by a patient who has taken a pain reliever for fewer than 7 consecutive days cannot be the result of the pain reliever.

People who are sensitive to one type of pain reliever should not attempt to use a different pain reliever.

A physician should not advise any patient to take any pain reliever for a period of longer than 7 consecutive days.

At least some people who take pain relievers for longer than 7 days will experience side effects.

Correct answer:

At least some people who take pain relievers for longer than 7 days will experience side effects.

Explanation:

This is an Inference question. In isolation, the phrase in the question stem, “most strongly supports”, could hint at either a Strengthen question or an Inference question. However, when we take the stem in its entirety, the structure of the problem begins to unfold. Remember: premises always support conclusions. Thus, if the information in the answer choices supports the argument above, the answer choices must be premises and the conclusion is found in the argument (leading us to believe the problem is a Strengthen question.) On the other hand, if the information in the body of the question supports the answer choices below, the argument’s conclusion must be found in the answer choices (leading us to believe the problem is an Inference question.) Since the question stem indicates that the information “above” is supporting answer choices below, the answer choices must be potential conclusions. This must be an Inference question.

Two primary insights can be gleaned reading the body of the question. First, since we are looking at an Inference question, our first line of defense is the “no new information” filter. Remember that valid conclusions must always (not just sometimes) be true, and therefore must be based entirely on the information found in the premises. Conclusions containing new information not found anywhere in the argument may or may not be true. The second insight is closely linked to the first. Throughout the entire body of the question, a lot of fuzzy, non-specific words are used: “most pain relievers”, “some people”, and “many people.” They describe subgroups of the total, and are very nebulous, especially when you contrast such phrases with “all pain relievers” or “all people.” Therefore, valid conclusions that go beyond these vague categorizations may or may not be true. Believing you can conclude something about “all people” when you only know about “some people” is a logical error known as overgeneralization. Once we recognize this trick of the Testmaker, it becomes relatively easy to spot many of the wrong answer choices.

Answer choice “A physician should not advise any patient to take any pain reliever for a period of longer than 7 consecutive days.” is a classic example of overgeneralization. Notice how this conclusion focuses on “any patient” taking “any pain reliever.” The body of the question only tells us about “most people” and “most pain relievers”. This conclusion goes beyond what we know, and therefore is not necessarily true.

Answer choice “People who are sensitive to one type of pain reliever should not attempt to use a different pain reliever.” includes new information not contained in the body of the question. The evidence in the top part of the question contains nothing about people being “sensitive” to one type of pain reliever. Therefore, we cannot make a conclusion about something we don’t have information on. Answer choice “People who are sensitive to one type of pain reliever should not attempt to use a different pain reliever.” is not necessarily true.

Answer choice “At least some people who take pain relievers for longer than 7 days will experience side effects. contains no new information, and remains within the fuzzy scope of the original statements. The body of the question tells us that “many people” who take painkillers for longer than 7 days experience side effects. The conclusion that “at least some people” will experience side effects is well within the information given. “At least some people who take pain relievers for longer than 7 days will experience side effects.” is basically a restate of information already given, so we can clearly conclude it must be true. “At least some people who take pain relievers for longer than 7 days will experience side effects.” is the right answer.

The conclusion in answer choice “Any side effects experienced by a patient who has taken a pain reliever for fewer than 7 consecutive days cannot be the result of the pain reliever.” also uses extreme scope limiters not justified by the original evidence. It refers to “any side effects” that “cannot” be the result of pain relievers. This goes well beyond the scope. The problem only tells us about side effects caused by a subset of pain relievers. It makes no mention of side effects not caused by pain relievers. Answer choice “Any side effects experienced by a patient who has taken a pain reliever for fewer than 7 consecutive days cannot be the result of the pain reliever.” fails the “no new information” filter. We cannot make a conclusion about something we don’t have information on. Answer choice “Any side effects experienced by a patient who has taken a pain reliever for fewer than 7 consecutive days cannot be the result of the pain reliever.” is not necessarily true.

Answer choice “Anyone who wants to maximize their natural health and well-being should avoid pain relievers entirely.” contains all sorts of new information not contained in the original evidence. The body of the question makes no mention on how to “maximize your natural health and well-being”, nor does it give us any criteria for when we should avoid pain relievers. (For example, could it be possible that the advantages of taking pain relievers could outweigh the side effects, even if we had to deal with these negative consequences?) “Anyone who wants to maximize their natural health and well-being should avoid pain relievers entirely.” cannot be a valid conclusion.

Example Question #2 : Standard Inference Logic

The price of health insurance as a percentage of an individual’s overall monthly income does not necessarily indicate quality of care. If it did, individuals who spent a greater percentage of their income on health insurance would receive better quality of care, or vice versa.

If the statements above are all true, which of the following can be properly inferred on the basis of them?

Possible Answers:

Reducing an individual’s spending on health insurance as a percent of their income will not necessarily lead to lower quality of care.

Individuals who spend the greatest percentage of their income on health insurance never receive high quality of care from medical providers.

It is probable that individuals who spend more on health insurance in fact get a lower quality of care than do those who spend less on health care.

If individuals receiving free or reduced cost health insurance from the government were removed from the sample, there would be a strong correlation between cost and quality of care.

Looking at the dollar amount spent on health insurance rather than the percentage would show a correlation between amount of money spent and quality of care.

 

 

 


Correct answer:

Reducing an individual’s spending on health insurance as a percent of their income will not necessarily lead to lower quality of care.

Explanation:

Whenever a GMAT question asks for something that can be "properly inferred" from a critical reasoning argument, remember that your job is to fully understand the argument presented and then look for the answer choice that is guaranteed by the information presented. Remember, the information doesn't have to be interesting - it just needs to be something that must be true given the information presented.

In this case, you are told there's no correlation between the percentage of an individual's income spent on healthcare and the quality of healthcare they receive. (Further, if it did exist, the argument states that either individuals who spend a greater percentage of their income on health insurance would get better or worse quality of care.)

The only real information that you have here is the fact that increased (or decreased) spending on health insurance as a percentage of income does not "necessarily indicate quality of care." You don't have any information on absolute amounts spent on health insurance or about the health insurance itself. From this, you can eliminate "If individuals receiving free or reduced cost health insurance from the government were removed from the sample, there would be a strong correlation between cost and quality of care.", which deals with eliminating a particular type of health insurance from the sample and "Looking at the dollar amount spent on health insurance rather than the percentage would show a correlation between amount of money spent and quality of care.", which deals with absolute dollar amounts. Choice "It is probable that individuals who spend more on health insurance in fact get a lower quality of care than do those who spend less on health care." can also be eliminated since you are told that there is no correlation in either direction.

Between "Individuals who spend the greatest percentage of their income on health insurance never receive high quality of care from medical providers." and "Reducing an individual’s spending on health insurance as a percent of their income will not necessarily lead to lower quality of care.", choice "Individuals who spend the greatest percentage of their income on health insurance never receive high quality of care from medical providers." can be eliminated since there is just no way to prove that individuals who spend a large percentage of their income on health insurance never get quality care from medical providers - the entire point of the argument is that there is no correlation.

Choice "Reducing an individual’s spending on health insurance as a percent of their income will not necessarily lead to lower quality of care." must be correct. If there is no correlation between spending on health insurance as a percentage of income, then reducing spending on health insurance as a percent of income may or may not affect quality of care. The words here "will not necessarily lead to lower quality of care" are particularly important, since it links back to the argument that there is just no way to tell.

Example Question #3 : Standard Inference Logic

When the defendant in a trial chooses not testify, the jury is not supposed to view this as evidence of the defendant’s guilt or innocence. Rather, the jury should base its decision on the evidence that is presented throughout the trial. Nevertheless, jurors will often take the failure of a defendant to testify as evidence of that defendant’s guilt.

Which of the following conclusions can most properly be drawn from the information above?

Possible Answers:

A defendant would sometimes be better served by testifying at trial rather than by choosing not to testify.

Most defendants who refuse to testify in their trials are, in fact, guilty.

The fact that a defendant refuses to testify is sometimes unfairly considered by a jury as evidence of guilt.

Some jurors refuse to take a defendant’s refusal to testify into consideration when deciding guilt or innocence.

The rules should be modified to require defendants to testify regardless of their guilt or innocence.

Correct answer:

The fact that a defendant refuses to testify is sometimes unfairly considered by a jury as evidence of guilt.

Explanation:

This is an inference question. The correct answer must be true. From the stimulus we know that the jury is not supposed to consider the fact that a defendant does not testify as evidence of guilt or innocence. And yet, jurors do take the refusal to testify as evidence of guilt. This means that choice "The fact that a defendant refuses to testify is sometimes unfairly considered by a jury as evidence of guilt." must be true, the fact that a defendant refuses to testify is unfairly seen by some jurors as evidence of guilt.

Choice "Most defendants who refuse to testify in their trials are, in fact, guilty." could be false, there is no indication of whether those who refuse to testify are guilty. Choice "The rules should be modified to require defendants to testify regardless of their guilt or innocence." indicates what “should” happen. This is a recommendation and is not something that must be true, even if it does seem logical. Choice "A defendant would sometimes be better served by testifying at trial rather than by choosing not to testify." could be true, a defendant might be better served by testifying, but it could be false as well. This is a prediction and it is very difficult for a prediction to be must be true. Choice "Some jurors refuse to take a defendant’s refusal to testify into consideration when deciding guilt or innocence." seems very plausible. Jurors are not supposed to take a defendant’s not testifying into account. It is logical to think that at least some jurors would follow this rule. However, it is not clear that this is the case. This does not reach the standard of must be true. The stimulus still allows the possibility that every juror takes the fact of not testifying into account.

Example Question #11 : Inference Critical Reasoning

About one-quarter of 'Top 50' business schools in the United States have acceptance rates of over 30 percent. Because of the higher acceptance rate, students admitted to these programs tend to have GMAT scores under 650, undergraduate grade point averages below 3.4, and work experience of less than four years.

Which of the following can be inferred from the passage above?

Possible Answers:

Most students whose GMAT scores are below 650 tend to have undergraduate grade point averages under 3.4.

Accepting over 30 percent of applicants can help a business school move into the 'Top 50'.

'Top 50' business schools tend to accept students with undergraduate grade point averages under 3.4.

Work experience is not the most important criterion for admission to a 'Top 50' business school.

It is possible for a business school whose accepted students have average GMAT scores under 650 to be 'Top 50'.

Correct answer:

It is possible for a business school whose accepted students have average GMAT scores under 650 to be 'Top 50'.

Explanation:

This inference question makes you choose between a guaranteed (and almost boring) answer choice and answer choices that tend to over-generalize from the information given in the prompt. As with any inference question, it is important to pay careful attention to word play within the answers. In this case, it forces you to differentiate between two groups: ‘Top 50’ business schools in general and the subset of ‘Top 50’ business schools discussed in the prompt.

For answer choice "Accepting over 30 percent of applicants can help a business school move into the 'Top 50'.", raising the acceptance rate has nothing to do with the quality of the school or its applicants. The argument doesn’t give any information about what it takes to become a top 50 School – eliminate "Accepting over 30 percent of applicants can help a business school move into the 'Top 50'.".

Although "Most students whose GMAT scores are below 650 tend to have undergraduate grade point averages under 3.4." is tempting, the argument does not make a connection between all students whose GMAT scores are below 650 and who have grade point averages under 3.4. While this connection might be true for the students admitted to the schools discussed, the argument doesn’t address GMAT takers in general, so this inference is not guaranteed.

Similarly, answer choice "'Top 50' business schools tend to accept students with undergraduate grade point averages under 3.4." over generalizes and tries to extrapolate general tendencies among all ‘Top 50’ business schools from the information given about one fourth of that group. Because there is no indication that you can extrapolate from the information given, "'Top 50' business schools tend to accept students with undergraduate grade point averages under 3.4." is not a proper inference.

Answer choice "Work experience is not the most important criterion for admission to a 'Top 50' business school." also makes the mistake of over-generalization. While some ‘Top 50’ schools admit students without much work experience, that does not mean that work experience isn’t the most important criterion for any ‘Top 50’ business school, as suggested in "Work experience is not the most important criterion for admission to a 'Top 50' business school.'.

Answer choice "It is possible for a business school whose accepted students have average GMAT scores under 650 to be 'Top 50'." is almost guaranteed by the argument. If admitted students at 25% of the ‘Top 50’ business schools tend to have scores around 650, it is possible that the average for those schools will be around 650. Since "It is possible for a business school whose accepted students have average GMAT scores under 650 to be 'Top 50'." only requires that it be possible, it is a proper inference and is the correct answer.

Example Question #1 : Standard Inference Logic

The continual use of chemical sprays in an effort to rid a house of insects has two unintended results that are particularly dangerous. First, chemical sprays often kill spiders, which are natural predators of most other insects found in the house. Second, chemical sprays often give rise to insects that are largely resistant to the sprays, since those insects that survive a particular spray will be the ones that are most resistant to the spray and will most prolifically breed spray-resistant offspring.

From the passage above, it can be inferred that the usefulness of chemical sprays can be improved by doing which of the following, assuming each is realistically possible?

Possible Answers:

Increasing the amount that is sprayed

Using only sprays that are more chemically stable

Spraying only a portion of the house at a time

Planting trees outside of the house to attract certain insects that typically are resistant to chemical sprays

Alternating the use of a variety of chemical sprays used

Correct answer:

Alternating the use of a variety of chemical sprays used

Explanation:

The argument states that there are two unintended results of spraying chemicals in the home. Chemicals kill spiders, which would normally kill other insects found in the home. Second, insects become resistant to the chemicals over the course of generations. The correct answer will solve (or at least mitigate) one of these two problems, thus increasing the effectiveness of the chemical sprays (the goal of the question). Remember that for any inference question that inferences must be guaranteed. This may mean that they are not interesting and may add information that seems almost identical to what is given in the argument.

Remember that it’s often to find what is wrong with an answer choice than finding what works about it. Answer choice "Using only sprays that are more chemically stable" doesn’t address anything to do with the spiders or insects becoming resistant, so it can be eliminated. Alternating the chemical spray used "Alternating the use of a variety of chemical sprays used" does address the insects becoming resistant. This will improve the effectiveness of the chemical sprays, so "Alternating the use of a variety of chemical sprays used" is the correct answer.

For completion, make sure that "Increasing the amount that is sprayed", "Spraying only a portion of the house at a time", and "Planting trees outside of the house to attract certain insects that typically are resistant to chemical sprays" can be eliminated. "Increasing the amount that is sprayed" can be eliminated because increasing the amount sprayed doesn’t address the problem of resistance and may make more spiders die. "Spraying only a portion of the house at a time" can be eliminated since, while spraying one portion of the house at a time might seem like it might address the issue of resistance, it doesn’t. Planting trees that house resistant insects "Planting trees outside of the house to attract certain insects that typically are resistant to chemical sprays" would be counterproductive.

Example Question #1 : Standard Inference Logic

Fretter Appliances sold more refrigerators in 2015 than in any previous year, and most of the refrigerators it sold that year were purchased by residents of Oakland County.  However, most refrigerators purchased by residents of Oakland County in 2015 were not purchased from Fretter Appliances.

Which of the following conclusions can be logically drawn from the statements above?

Possible Answers:

In 2015, more refrigerators were purchased by residents of Oakland County than were sold by Fretter Appliances.

At least some residents of Oakland County purchased refrigerators from stores not located within Oakland County in 2015.

At least one store in Oakland County sold more refrigerators in 2015 than Fretter Appliances did.

Fretter Appliances sold more refrigerators to residents of Oakland County in 2015 than it did in any previous year.

Residents of Oakland County purchased more refrigerators in 2015 than in any previous year.

Correct answer:

In 2015, more refrigerators were purchased by residents of Oakland County than were sold by Fretter Appliances.

Explanation:

This Inference problem forces you to deal with the provided statistics, which guarantee that choice B must be true. To prove that, you could use a variable for the number of refrigerators that Fretter sold to residents of Oakland County (let's call it x) or you can borrow a tool from your Word Problems / Quantitative toolkit and pick a number (such as 50) .

You know that most (so > 1/2) of the refrigerators that Fretter sold were to residents of Oakland County. So the number of refrigerators that Fretter sold in total must be less than 2x, or less than 100.

You also know that more than 1/2 of the refrigerators sold to residents of Oakland County were NOT from Fretter. So Fretter's x (or 50) refrigerators are less than half of Oakland County's refrigerator sales. Oakland County's sales then are > 2x, or > 100. This allows you to directly compare the two totals: The number of total Fretter sales is less than the number of total Oakland County sales. Choice "In 2015, more refrigerators were purchased by residents of Oakland County than were sold by Fretter Appliances." is therefore proven.

Among the incorrect answer choices:

With choice "At least one store in Oakland County sold more refrigerators in 2015 than Fretter Appliances did.", recognize that no one store had to sell more in Oakland County than Fretter in order for Fretter's sales to be less than 50% of the county's. Several smaller stores could add up to that >50% amount.

With choice "Residents of Oakland County purchased more refrigerators in 2015 than in any previous year.", there is just no information to draw this conclusion: Fretter's 2015 was its greatest ever, but you don't have any information about Oakland County's historical sales.

Choice "Fretter Appliances sold more refrigerators to residents of Oakland County in 2015 than it did in any previous year." can be eliminated by considering extreme cases when picking numbers. If this year Fretter sold 100 refrigerators overall (its best year ever) and 51 in Oakland County (more than half its total sales), you could still have Fretter selling 90 last year (consistent with 2015 as its highest ever sales) with all 90 of them coming from Oakland County (just a much higher percentage of its sales coming from Oakland County in a previous year).

And choice "At least some residents of Oakland County purchased refrigerators from stores not located within Oakland County in 2015." is a choice that seems likely to be true, but that has no proof anywhere in the stimulus. On Inference questions, if you can't find direct proof, the answer choice is not necessarily true and must be eliminated.

Example Question #7 : Standard Inference Logic

The National Academy of Sciences (NAS) recommends a specific daily intake for vitamin C, as greatly exceeding that amount is dangerous. Many vitamin-fortified foods contain 100% of this recommended daily intake for vitamin C in one serving, an amount defined on the package by the manufacturer. However, most consumers overestimate the amount of one serving for these foods, ingesting two to four times what is considered one serving by the manufacturer.

Which of the following is most supported by the information above?

Possible Answers:

Manufacturers need to change the amount listed as one serving on the packaging for vitamin-fortified foods.

People should avoid taking supplemental vitamin C if they are eating vitamin-fortified foods.

Most people eating vitamin-fortified foods are consuming dangerous amounts of vitamin C.

Some people eating vitamin-fortified foods exceed the daily intake for vitamin C that is recommended by the National Academy of Sciences.

Any person eating vitamin-fortified foods will receive the daily intake for vitamin C that is recommended by the National Academy of Sciences.

Correct answer:

Some people eating vitamin-fortified foods exceed the daily intake for vitamin C that is recommended by the National Academy of Sciences.

Explanation:

This question stem is asking you to form a conclusion based on the information given, so you must pick the one answer that is guaranteed. As is true for all inference style questions, you should use process of elimination by evaluating each potential conclusion.

For "Most people eating vitamin-fortified foods are consuming dangerous amounts of vitamin C.", you do know that “most consumers overestimate the amount of one serving for these foods, ingesting two to four times what is considered one serving by the manufacturer” so it is safe to say that most people get more than their daily intake as recommended by the NAS. However, to be dangerous the recommended amounts must be “greatly exceeded” and we have no idea if “two to four times” meets that threshold. As a result this is not a proper inference.

For "Manufacturers need to change the amount listed as one serving on the packaging for vitamin-fortified foods.", there is no proof given in the stimulus that manufacturers need to do anything. While it is true that many consumers overestimate the amount of one serving, this does not allow you to conclude that manufacturers need to make a change (maybe the consumers just need to get better at estimating!). This type of prescription is virtually impossible to prove in an inference style question.

For "Any person eating vitamin-fortified foods will receive the daily intake for vitamin C that is recommended by the National Academy of Sciences.", the word “any” makes this easy to eliminate. We know that many, but not necessarily all, vitamin fortified foods contain 100% of the recommended vitamin C and that “most” consumers overestimate a serving. However, this still leaves open the possibility that some people are eating vitamin-fortified foods that do not contain vitamin C or that they are not getting a full serving’s worth.

For "Some people eating vitamin-fortified foods exceed the daily intake for vitamin C that is recommended by the National Academy of Sciences.", you know with certainty that most consumers are eating 2-4 servings of vitamin-fortified foods containing vitamin C, which provides more than 100% of the recommended amount. Since you only need to prove one person has consumed more than a serving of these foods to be sure of this conclusion, it must be true and "Some people eating vitamin-fortified foods exceed the daily intake for vitamin C that is recommended by the National Academy of Sciences." is correct.

For "People should avoid taking supplemental vitamin C if they are eating vitamin-fortified foods.", this is a similar prescription to what you saw in "Manufacturers need to change the amount listed as one serving on the packaging for vitamin-fortified foods.". There might be many reasons why someone wants or needs to take a vitamin C supplement even if they are eating the vitamin-fortified foods. Maybe their doctor wants them to have lots of extra vitamin C or maybe they are eating the vitamin-fortified foods that do not contain 100% of the recommended amount. This is not a proper inference.

Tired of practice problems?

Try live online GMAT prep today.

1-on-1 Tutoring
Live Online Class
1-on-1 + Class
Learning Tools by Varsity Tutors